Question about Lipschitz continuity

Click For Summary
The discussion revolves around proving that if a function f is locally Lipschitz continuous on a normed linear space X and K is a compact set in X, then there exists an open set U containing K where f is Lipschitz continuous. The approach involves using an open cover of K with sets where f is Lipschitz and extracting a finite subcover. The idea of replacing K with its convex hull is proposed to maintain compactness while ensuring the final open set is convex. The solution concludes that selecting an open set slightly larger than K, yet still convex and within the subcover, successfully demonstrates the required Lipschitz continuity. This method effectively addresses the problem posed in the homework statement.
quasar987
Science Advisor
Homework Helper
Gold Member
Messages
4,796
Reaction score
32

Homework Statement


This should be easy but I'm stomped.

Let K be a compact set in a normed linear space X and let f:X-->X be locally Lipschitz continuous on X. Show that there is an open set U containing K on which f is Lipschitz continuous.

Homework Equations


locally Lipschitz means that for every x in X, there is a nbdh around x on which f is Lipschitz.

The Attempt at a Solution


The obvious thing to do it seems it take an open cover of K by sets on which f is Lipschitz continuous and extract a finite subcover. But then what?!?
 
Physics news on Phys.org
Wouldn't it be nice if you could make sure that the final open set containing K and covered by the subcover, were convex? Replace K by it's convex hull, it's still compact, right? Then take the finite subcover. Then pick a open set just a 'little' bigger than K, but still convex and contained in the subcover. That works, doesn't it?
 
Last edited:
Of course, yes! Thanks Dick.
 
Question: A clock's minute hand has length 4 and its hour hand has length 3. What is the distance between the tips at the moment when it is increasing most rapidly?(Putnam Exam Question) Answer: Making assumption that both the hands moves at constant angular velocities, the answer is ## \sqrt{7} .## But don't you think this assumption is somewhat doubtful and wrong?

Similar threads

Replies
4
Views
8K
Replies
23
Views
3K
  • · Replies 4 ·
Replies
4
Views
2K
  • · Replies 9 ·
Replies
9
Views
4K
Replies
12
Views
2K
  • · Replies 12 ·
Replies
12
Views
2K
  • · Replies 1 ·
Replies
1
Views
2K
  • · Replies 3 ·
Replies
3
Views
3K
Replies
20
Views
4K
Replies
9
Views
3K